Warum können wir den Endpunkt bei der Ableitung der Euler-Lagrange-Gleichung in der Mechanik als fest betrachten?

In der Mechanik erhält man die Bewegungsgleichungen (Euler-Lagrange-Gleichungen) über das Hamiltonsche Prinzip, indem man stationäre Punkte der Wirkung betrachtet

S = T ich T F L   D T
wo wir haben L = T v , die Differenz zwischen kinetischer und potentieller Energie. Die übliche Ableitung setzt die erste Variation auf Null und integriert partiell, um die Anforderung zu erhalten
δ S = T ich T F [ L Q D D T ( L Q ˙ ) ] δ Q   D T + L Q ˙ ( T F )   δ Q ( T F ) L Q ˙ ( T ich )   δ Q ( T ich ) = 0
Wo Q bezeichnet die verallgemeinerten Koordinaten und Q ˙ die entsprechenden Geschwindigkeiten.

An dieser Stelle eliminieren die meisten Lehrbuchableitungen den zweiten und dritten Term durch Behauptung δ Q ( T ich ) = 0 Und δ Q ( T F ) = 0 . Die erste davon ist intuitiv, weil wir in der Praxis normalerweise Anfangswertprobleme betrachten, bei denen die Anfangspositionen bekannt sind. Aber a priori wissen wir es normalerweise nicht Q ( T F ) für eine beliebige Zeit T F , also warum setzen wir δ Q ( T F ) = 0 ?

Bei einigen anderen Variationsprinzipien ist es intuitiv anzunehmen, dass die Koordinaten an beiden Endpunkten bekannt und fest sind, zum Beispiel das Fermat-Prinzip, um den Weg eines Lichtstrahls zwischen zwei Punkten zu berechnen. Gibt es eine intuitive Erklärung dafür, warum die Endkoordinaten bei Anwendung des Hamilton-Prinzips als fest angesehen werden, oder eine Ableitung der mechanischen Euler-Lagrange-Gleichungen ohne diese Annahme?

Indem ich das Problem selbst betrachtete, versuchte ich, die gleichen Bedingungen auf andere Weise zu erhalten: wenn wir stattdessen die endgültige Position einnehmen Q ( T F ) wie kostenlos, aber mit T F fest, dann erhalten wir zusätzlich zur Euler-Lagrange-Gleichung die zusätzliche Anforderung für die Stationarität

L Q ˙ ( T F ) = 0
aber das scheint im allgemeinen nicht zu gelten. Betrachten wir beispielsweise einen harmonischen Oszillator, impliziert diese Bedingung, dass die kinetische Energie zum (beliebig) festgelegten Zeitpunkt minimiert wird T F . Ich habe die notwendigen Bedingungen noch nicht berücksichtigt, wenn wir dies auch berücksichtigen T F so frei, da ich mir nicht ganz sicher bin, wie ich die Analyse durchführen soll, ohne Elemente aus der Theorie der optimalen Steuerung (zB das Pontryaginsche Prinzip oder die HJB-Gleichung) einzubeziehen.

Im Wesentlichen ein Duplikat von physical.stackexchange.com/q/38348/2451 , Physics.stackexchange.com /q/245704/2451 , Physics.stackexchange.com /q/161586/2451 und Links darin.
Danke, ich habe es nicht geschafft, diese Fragen zu finden, als ich früher gesucht habe, und die Antworten darauf sind ziemlich interessant. Ich konnte jedoch keine Ableitung der EL-Gleichungen in denen finden, bei denen der Endpunkt als frei angesehen wird (was der zweite Teil meiner Frage war), obwohl die Kommentare auf physical.stackexchange.com/questions/38348 erwähnen, dass dies der Fall ist möglich.

Antworten (2)

  1. Normalerweise wird uns in der Physik ein Problem gegeben, zB ein Anfangswertproblem (IVP) oder ein Randwertproblem (BVP)? Diese beiden Arten von Problemen sollten nicht miteinander vermischt werden, vgl. zB this , this & this Phys.SE Beiträge.

  2. Bei dynamischen (im Gegensatz zu statischen) Problemen sind manchmal ein stationäres Aktionsprinzip oder ein Maupertuis-Prinzip/abgekürztes Aktionsprinzip für BVPs möglich, aber niemals für IVPs, wenn wir Lokalität benötigen 1 .

  3. Für das stationäre Wirkungsprinzip bestehen einige mathematische Freiheiten bei der Wahl konsistenter Randbedingungen (BCs), vgl. zB meine Math.SE-Antwort hier . Die Physik schreibt jedoch oft vor, welche BCs relevant sind.

--

1 Es existieren verschiedene nicht-lokale Wirkungsformulierungen für IVPs, zB die bilokale Gurtin-Tonti-Methode, vgl. dieser Phys.SE-Beitrag.

Vielen Dank! Die Punkte 1. und 3. sind ziemlich klar, und insbesondere 1. hat mir klar gemacht, dass wir bei der Anwendung des Hamilton-Prinzips wirklich eine BVP in Betracht ziehen, die Bewegungsgleichungen jedoch für IVPs desselben Systems dieselben sind, sodass wir sie immer noch finden können sie mit Euler-Lagrange. Haben Sie eine Referenz oder eine Begründung für 2. ? Mein ursprünglicher Gedanke war, dass vielleicht eine Variationsformulierung für eine IVP möglich ist, indem zugelassen wird, dass die letzte Zeit frei oder unendlich ist.
Vielleicht verstehe ich 2. falsch, aber wenn wir ein IVP für das System erster Ordnung in Betracht ziehen X ˙ = X mit X ( 0 ) = X 0 , ist die Lösung durch den Minimierer von gegeben 0 ( X 2 + X ˙ 2 )   D T mit X ( 0 ) = X 0 fest, was wir überprüfen können, indem wir eine 'Eingabe'-Variable einführen u = X ˙ und Anwendung des standardmäßigen linear-quadratischen Reglers (LQR) aus der Steuerungstheorie. Ist das nicht ein (lokales) Mindestprinzip für diese IVP?

Wenn Fermats Prinzip für Sie intuitiv ist, ist Hamiltons Prinzip nicht viel anders. Beide sagen im Grunde aus, dass sich das System (oder Licht) zwischen zwei Fixpunkten so bewegt, dass die Aktion (oder Zeit) maximal/minimal ist.

Ich bin mir nicht sicher, wie intuitiv das ist. Natürlicher ist es für die meisten Menschen wahrscheinlich, an die zeitliche Entwicklung von Systemen zu denken, dh man bereitet das System in einem bestimmten Zustand vor (oder schickt den Lichtstrahl in eine bestimmte Richtung) und sieht, was passiert, d.h. wo es endet.

Die Prinzipien von Hamilton/Fermat sind gut, weil sie allgemein sind (was Physikern gefällt).

Zu Ihrer Frage, warum setzen wir δq(tf)=0δq(tf)=0? , das ist im Grunde das, was das Hamilton-Prinzip besagt. Mit anderen Worten: Sie betrachten alle möglichen Wege von einem Anfangspunkt zu einem Endpunkt und das System nimmt den idealen Weg. Sie betrachten nicht alle möglichen Pfade zwischen allen möglichen Punkten.

Nehmen Sie als reales Beispiel als Ausgangspunkt Ihr Zuhause und als Zielpunkt Ihren Arbeitsplatz. Es gibt alle möglichen Wege, die Sie zwischen ihnen gehen könnten. Sie (oder "Natur", wenn Sie möchten) entscheiden sich jedoch für einen einzigen Weg, der ideal ist. Abhängig von Ihren Prioritäten (= Aktionsfunktional) kann dies der Weg sein, der die kürzeste Zeit benötigt, oder der Weg, der am billigsten ist usw.

Beachten Sie, dass Sie zum Finden dieses idealen Pfads nicht die Pfade von Ihrem Zuhause zum Schwimmbad oder die Pfade zwischen der Arbeit und dem Flughafen usw. berücksichtigen.

Danke, Ihr erster Absatz (zusammen mit der Antwort von QMechanic) hat mir klar gemacht, dass sich das Hamilton-Prinzip wirklich auf ein Grenzwertproblem bezieht und dass das Anfangswertproblem, das ich als "intuitiver" betrachtete, zufällig dieselbe Bewegungsgleichung hat. dh die Euler-Lagrange-Gleichungen.